[Tex/LaTex] Exam package, award points by part, but droppoints by question

exam

(MWE at the bottom) I am using the exam package. I had the following basic layout for my questions

\question[10] question 1
    \begin{parts}
    \part part a
    \part part b
    \end{parts}
    \droppoints

which was working fine and dropping the points where I wanted them — at the end of the question, bottom of the page

I decided to award points by parts, but I still want the point total after the whole question. But I'm being told \droppoints is an "undefined control sequence" when I try moving the point values to the parts

\question question1
\begin{parts}
\part[5] part a
\part[5] part b
\end{parts}
\droppoints

I cannot find what's wrong (and yes, I tried the documentation).

Note that I also have these settings (which I doubt would affect this)

\pointsdroppedatright
\bracketedpoints
\pointpoints{pt}{pts}
\marginpointname{ pts}

Here's an MWE

\documentclass[fleqn,addpoints,answers]{exam}
%\usepackage{amsfonts,amsmath,amssymb,amsthm,mathtools,commath}

% For the exam package
% POINT DISPLAY
\pointsdroppedatright   % Display points lower right
\bracketedpoints        % square bracket around points
\pointpoints{pt}{pts}
\marginpointname{ pts}

\begin{document}
\begin{questions}

\question%[10]
Question 1
\begin{parts}
  \part[5]
    Calculate the future value annuity factor $s_{n|i}$.
    \part[5]
    Use your answer from (a) to find the future value.
\end{parts}
\droppoints

\end{questions}
\end{document}

Best Answer

(MWE at Bottom)

The \droppoints commands takes the optional parameter of \question (or \part, see the question 3 below).

If you use parts and you want to get the sum of all parts, then you must use \droptotalpoints.

If you want to see also the points for each part, then you may use the \droppoints inside the parts-environment.

enter image description here

\documentclass[fleqn,addpoints,answers]{exam}
%\usepackage{amsfonts,amsmath,amssymb,amsthm,mathtools,commath}

% For the exam package
% POINT DISPLAY
\pointsdroppedatright   % Display points lower right
\bracketedpoints        % square bracket around points
\pointpoints{pt}{pts}
\marginpointname{ pts}

\begin{document}
\begin{questions}

\question
Question 1
\begin{parts}
  \part[5]
    Calculate the future value annuity factor $s_{n|i}$.
    \part[5]
    Use your answer from (a) to find the future value.
\end{parts}
\droptotalpoints


\question[12]
Question 1
\begin{parts}
  \part[5]
    Calculate the future value annuity factor $s_{n|i}$.
    \part[5]
    Use your answer from (a) to find the future value.
\end{parts}
\droppoints


\question
Question 1
\begin{parts}
  \part[7]
    Calculate the future value annuity factor $s_{n|i}$.
\droppoints
    \part[7]
    Use your answer from (a) to find the future value.
\droppoints
\end{parts}
\droptotalpoints


\end{questions}

\end{document}
Related Question